Derivación de la relación anticonmutación entre operadores de creación/aniquilación para fermiones de Dirac

A partir de los campos de Dirac:

Ψ ( X ) = 1 ( 2 π ) 3 / 2 d 3 k 2 ω k r [ C r ( k ) tu r ( k ) mi i k X + d r ( k ) v r ( k ) mi i k X ] k 0 = ω k

Ψ ( X ) = 1 ( 2 π ) 3 / 2 d 3 k 2 ω k r [ d r ( k ) v r ( k ) mi i k X + C r ( k ) tu r ( k ) mi i k X ] k 0 = ω k

dónde ω k = k 2 + metro 2 .

La condición de quatización canónica dice:

{ { Ψ α ( X ) , Ψ β ( y ) } t = d ( 3 ) ( X y )     d α β { Ψ α ( X ) , Ψ β ( y ) } t = 0 { Ψ α ( X ) , Ψ β ( y ) } t = 0

Para derivar la condición de cuantización para los operadores de creación/aniquilación, tengo que reescribir C , C , d , d en términos de Ψ y Ψ .

Por ejemplo, para derivar la condición de cuantificación canónica entre C , C Puedo reescribirlos como:

C r ( k ) = 1 2 π 3 d 3 X 2 ω k tu r ( k ) Ψ ( X ) mi i k X

C s ( pag ) = 1 2 π 3 d 3 y 2 ω pag Ψ ( y ) tu s ( pag ) mi i pag y

y luego calcule explícitamente el anti-conmutador:

{ C r ( k ) , C s ( pag ) } t = 1 ( 2 π ) 3 d 3 X d 3 y 2 ω k 2 ω pag [ tu r ( k ) Ψ ( X ) Ψ ( y ) tu s ( pag ) + Ψ ( y ) tu s ( pag ) tu r ( k ) Ψ ( X ) ] mi i ( k X pag y ) = 1 ( 2 π ) 3 d 3 X d 3 y 2 ω k 2 ω pag [ tu r ( k ) { Ψ ( X ) , Ψ ( y ) } tu s ( pag ) ] mi i ( k X pag y ) =

Pero aquí me falta algo: no entiendo por qué puedo cambiar tu s ( pag ) y tu r ( k ) en el segundo término para recuperar el anticonmutador entre Ψ y Ψ .

En todos los libros de texto sagrados es al revés. Usted postula las relaciones de anticonmutación para C r y d r y luego, está a un simple paso de derivar las relaciones de anticonmutación para los campos.

Respuestas (2)

Debe proceder en contracción de los índices de espinor y recordar que, por ejemplo, un par tu ( k , r ) Ψ ( X ) tu α ( k , r ) Ψ α ( X ) , con tu α ( k , r ) etiquetando el α el componente del espinor de Dirac tu ( k , r ) . Por lo tanto,

{ C ( k , r ) , C ( pag , s ) } d 3 X d 3 y ( tu α ( k , r ) Ψ α ( X ) Ψ β ( y ) tu β ( pag , s ) + Ψ β ( y ) tu β ( pag , s ) tu α ( k , r ) Ψ α ( X ) ) = d 3 X d 3 y ( tu α ( k , r ) { Ψ α ( X ) , Ψ β ( y ) } tu β ( pag , s ) ) = =

Termine el ejercicio utilizando sus anticonmutadores canónicos de campo de cuantificación junto con las relaciones de ortogonalidad/completitud entre los tu 's.

Aquí Ψ ( X ) es una matriz columna que tiene 4 componentes y Ψ ( X ) es una matriz de fila con 4 elementos de fila (por supuesto, estos elementos son funciones de X ).

Y cuando está tomando la anticonmutación, está eligiendo un componente (o elemento) Ψ α ( X ) de (4 × 1) matriz de columnas Ψ ( X ) y de manera similar, debe elegir un componente Ψ β ( X ) de 1 × 4) matriz de filas Ψ ( X ) .

Ψ α ( X ) = 1 ( 2 π ) 3 / 2 d 3 k 2 ω k r = 1 , 2 [ C r ( k ) tu r , α ( k ) mi i k X + d r ( k ) v r , α ( k ) mi i k X ] k 0 = ω k Ψ β ( X ) = 1 ( 2 π ) 3 / 2 d 3 k 2 ω k r = 1 , 2 [ d r ( k ) v r , β ( k ) mi i k X + C r ( k ) tu r , β ( k ) mi i k X ] k 0 = ω k
Entonces, ahora puedes ver allí tu y v tiene dos índices r y α (o β ), aquí r puede tomar los valores 1 y 2 mientras α (o β ) puede tomar valores 1,2,3,4.

O simplemente tu r , α (o v r , α ) es el α -th componente (o α -ésimo elemento de la matriz) de los (4 × 1) matriz de columnas tu r (o v r ).

Y de manera similar, tu r , α (o v r , α ) es el α -ésimo componente (o α -ésimo elemento de la matriz) del (1 × 4) matriz de filas tu r (o v r ).

De este modo tu r , α , v r , α , tu r , α , v r , α todos estos son solo números o los elementos de la matriz, no las matrices.

Por lo tanto, proceda de la forma en que iba y cambie fácilmente tu s , α ( pag ) y tu r , α ( k ) (en su notación) ya que son solo los números o los componentes (o elementos) de las matrices correspondientes.